Is z^c Analytic When a Branch is Chosen for Complex Numbers z and c?

  • Thread starter Thread starter Bashyboy
  • Start date Start date
  • Tags Tags
    Derivative
Click For Summary
The discussion centers on the analyticity of the complex function z^c when a branch is chosen. It is established that z^c is analytic in the domain defined by the chosen branch, as it can be expressed as e^(c log z), where log z incorporates the multivalued nature of the argument. By restricting the domain to eliminate ambiguity in the argument, such as excluding the negative real axis, z^c can be made single-valued. The derivative of z^c is derived using the chain rule, resulting in the formula d/dz z^c = c z^(c-1). The challenge remains in understanding how to generalize the removal of branches to ensure z^c remains single-valued.
Bashyboy
Messages
1,419
Reaction score
5
b1. Homework Statement
Let ##c## and ##z## denote complex numbers. Then

1. When a branch is chosen for ##z^c##, then ##z^c## is analytic in the domain determined by that branch.

2. ##\frac{d}{dz} z^c = c z^{c-1}##

Homework Equations

The Attempt at a Solution



In regards to number one, we have that ##z^c## is defined as ##z^c = e^{c \log z}## which can be written as ##z^c = e^{c(\ln |z| + i \arg z}##. For ##z^c## to be analytic, it must first be a function. In the textbook I am using, the author discussing how we can remove a branch of the domain of a multi-valued function##f(z)## so that it can be become a function.

For a given complex number, ##\ln|z|## will return a single value; however, ##\arg z## will return multiple values, whereby we get different values of ##z^c##. Thus, the problem of ##z^c## being multivalued lies with the fact that ##\arg z## is multivalued. So, if I were to restrict the domain so that ##\arg z## was unique, then ##z^c## would be single-valued.

I know that if I replaced ##\arg z## with ##Arg ~z##, then ##z^c## would be multivalued. So, if I restricted the domain by removing all of the complex numbers along the negative real axis, including zero, then what would happen to ##\arg z##?

How could I formulate a general restriction? I was thinking that if we remove the ray which makes an angle of ##\theta## with the positive real axis, all remove all those points, then the range of ##\arg z## would become ##(\theta - \pi, \pi + \theta)##. But I am unsure of how to justify this.
______________________________________________________________________________

Number two is giving me some difficulty. At my disposal, I have ##\frac{d}{dz} e^z = e^z## and the chain rule.

If I take the derivative of ##z^c = e^{c(\ln |z| + i \arg z}##, I would have to take the derivative of ##e^z##, which I already know, and ##c(\ln |z| + i \arg z)##, which I do not know. I would venture to guess that ## \frac{d}{dz} \ln |z| = \frac{1}{|z|}##, but this has not been established. What would the derivative of ##\arg z## be?
 
Physics news on Phys.org
Okay, I was able to figure out how to differentiate the function ##z^c## (of course, to be able to differentiate this function, we first have to remove a branch of the function, of which I am still uncertain of how to do). I found in my textbook that ##e^{\log z} = z##. Using the chain rule, I could compute the derivative and find out what the derivative of ##\log z## is, when you remove a branch from the function:

##
\frac{d}{dz} [e^{\log z}] = \frac{d}{dz} z \iff
##

##
\frac{d}{dz}[\log z] e^{\log z} = 1 \iff
##

Because ##e^{\log z}## is never zero, we can divide by this quantity to get

##
\frac{d}{dz} [\log z ] = \frac{1}{e^{\log z}} \iff
##

##
\frac{d}{dz} [\log z ] = \frac{1}{z}
##

Now we have the requisite knowledge to differentiate ##z^c##:

##
\frac{d}{dz} z^c = \frac{d}{dz} [e^{c \log z}] \iff
##

##
\frac{d}{dz} z^c = \frac{d}{dz} [e^{\log z^c}] \iff
##

##
\frac{d}{dz} z^c = \frac{d}{dz} [\log z^c] e^{\log z^c} \iff
##

##
\frac{d}{dz} z^c = c \frac{1}{z} e^{c \log z} \iff
##

##
\frac{d}{dz} z^c = c \frac{1}{z} z^c \iff
##

##
\frac{d}{dz} z^c = c z^{c-1}
##

So, this much I figured out. However, I am still having difficulty with this concept of removing a branch. In general, how do you remove an arbitrary branch from the function ##z^c##, so that it becomes a single-valued function, rather than a multi-valued one?
 
Last edited:
Question: A clock's minute hand has length 4 and its hour hand has length 3. What is the distance between the tips at the moment when it is increasing most rapidly?(Putnam Exam Question) Answer: Making assumption that both the hands moves at constant angular velocities, the answer is ## \sqrt{7} .## But don't you think this assumption is somewhat doubtful and wrong?

Similar threads

Replies
8
Views
3K
  • · Replies 5 ·
Replies
5
Views
2K
  • · Replies 4 ·
Replies
4
Views
3K
  • · Replies 6 ·
Replies
6
Views
2K
Replies
7
Views
2K
  • · Replies 13 ·
Replies
13
Views
3K
  • · Replies 12 ·
Replies
12
Views
5K
  • · Replies 3 ·
Replies
3
Views
2K
Replies
4
Views
2K
  • · Replies 6 ·
Replies
6
Views
2K